Difference between revisions of "2012 AMC 12B Problems/Problem 24"

(Solution)
(Solution: missed an important case)
Line 17: Line 17:
 
<math>f_1(3^4)=4^3=2^6</math> works, therefore any number <math>\leq 400</math> that are divisible by <math>3^4</math> works: there are <math>4</math> of them.
 
<math>f_1(3^4)=4^3=2^6</math> works, therefore any number <math>\leq 400</math> that are divisible by <math>3^4</math> works: there are <math>4</math> of them.
  
<math>3^3 \cdot Q^2</math> could also work if <math>Q</math> is odd, but <math>3^3 \cdot 5^2 >400</math> already.
+
<math>3^3 \cdot Q^2</math> could also work if <math>Q^2</math> satisfies <math>2~|~f_1(Q^2)</math>, but <math>3^3 \cdot 5^2 > 400</math>.
  
 
<math>f_1(5^3)=6^2 = 2^2 3^2</math> does not work.
 
<math>f_1(5^3)=6^2 = 2^2 3^2</math> does not work.
Line 23: Line 23:
 
<math>f_1(7^3)=8^2=2^6</math> works. There are no other multiples of <math>7^3</math> within <math>400</math>.
 
<math>f_1(7^3)=8^2=2^6</math> works. There are no other multiples of <math>7^3</math> within <math>400</math>.
  
<math>7^2 \cdot Q^3</math> could also work if <math>Q</math> is odd, but <math>7^2 \cdot 3^3 > 400</math> already.
+
<math>7^2 \cdot Q^2</math> could also work if <math>4~|~f_1(Q^2)</math>, but <math>7^2 \cdot 3^2 > 400</math> already.
  
<math>f_1(11^2 \cdot 3) = 12 \cdot 4 = 2^4 \cdot 3</math>, which does not work.
+
For number that are only divisible by <math>p=11, 13, 19</math>, they don't work because none of these primes are such that <math>p+1</math> could be a multiple of <math>2^5</math> nor a multiple of <math>3^4</math>.
  
For <math>p=13, 17, 19</math>, any odd multiple <math>p^2</math> other than itself is greater than <math>400</math>. Moreover, none of these primes are such that <math>p+1</math> could be a multiple of <math>2^5</math>.  
+
<math>p=17</math> works because <math>f_1(17^2) = 18=3^4 \cdot 2</math>.  
  
In conclusion, there are <math>12+4+1=17</math> number of <math>N</math>'s ... <math>\framebox{C}</math>.
+
In conclusion, there are <math>12+4+1+1=18</math> number of <math>N</math>'s ... <math>\framebox{D}</math>.

Revision as of 05:48, 6 December 2012

Problem 24

Define the function $f_1$ on the positive integers by setting $f_1(1)=1$ and if $n=p_1^{e_1}p_2^{e_2}\cdots p_k^{e_k}$ is the prime factorization of $n>1$, then \[f_1(n)=(p_1+1)^{e_1-1}(p_2+1)^{e_2-1}\cdots (p_k+1)^{e_k-1}.\] For every $m\ge 2$, let $f_m(n)=f_1(f_{m-1}(n))$. For how many $N$ in the range $1\le N\le 400$ is the sequence $(f_1(N),f_2(N),f_3(N),\dots )$ unbounded?

Note: A sequence of positive numbers is unbounded if for every integer $B$, there is a member of the sequence greater than $B$.

$\textbf{(A)}\ 15\qquad\textbf{(B)}\ 16\qquad\textbf{(C)}\ 17\qquad\textbf{(D)}\ 18\qquad\textbf{(E)}\ 19$


Solution

First of all, notice that for any odd prime $p$, the largest prime that divides $p+1$ is no larger than $\frac{p+1}{2}$, therefore eventually the factorization of $f_k(N)$ does not contain any prime larger than $3$. Also, note that $f_2(2^m) = f_1(3^{m-1})=2^{2m-4}$, when $m=4$ it stays the same but when $m\geq 5$ it grows indefinitely. Therefore any number $N$ that is divisible by $2^5$ or any number $N$ such that $f_k(N)$ is divisible by $2^5$ makes the sequence $(f_1(N),f_2(N),f_3(N),\dots )$ unbounded. There are $12$ multiples of $2^5$ within $400$. Now let's look at the other cases.

Any first power of prime in a prime factorization will not contribute the unboundedness because $f_1(p^1)=(p+1)^0=1$. At least a square of prime is to contribute. So we test primes that are less than $\sqrt{400}=20$:

$f_1(3^4)=4^3=2^6$ works, therefore any number $\leq 400$ that are divisible by $3^4$ works: there are $4$ of them.

$3^3 \cdot Q^2$ could also work if $Q^2$ satisfies $2~|~f_1(Q^2)$, but $3^3 \cdot 5^2 > 400$.

$f_1(5^3)=6^2 = 2^2 3^2$ does not work.

$f_1(7^3)=8^2=2^6$ works. There are no other multiples of $7^3$ within $400$.

$7^2 \cdot Q^2$ could also work if $4~|~f_1(Q^2)$, but $7^2 \cdot 3^2 > 400$ already.

For number that are only divisible by $p=11, 13, 19$, they don't work because none of these primes are such that $p+1$ could be a multiple of $2^5$ nor a multiple of $3^4$.

$p=17$ works because $f_1(17^2) = 18=3^4 \cdot 2$.

In conclusion, there are $12+4+1+1=18$ number of $N$'s ... $\framebox{D}$.